LSAT 2 – Section 2 – Question 10

You need a full course to see this video. Enroll now and get started in less than a minute.

Target time: 1:20

This is question data from the 7Sage LSAT Scorer. You can score your LSATs, track your results, and analyze your performance with pretty charts and vital statistics - all with a Free Account ← sign up in less than 10 seconds

Question
QuickView
Type Tags Answer
Choices
Curve Question
Difficulty
Psg/Game/S
Difficulty
Explanation
PT2 S2 Q10
+LR
Strengthen +Streng
A
35%
156
B
2%
151
C
56%
163
D
6%
148
E
2%
148
148
157
166
+Harder 144.588 +SubsectionEasier
This page shows a recording of a live class. We're working hard to create our standard, concise explanation videos for the questions in this PrepTest. Thank you for your patience!

We should recognize this is a strengthening question, since it asks: Which one of the following, if true, would most strengthen the argument?

Our stimulus begins by telling us that advertisers are often seen as unscrupulous (lacking restraint, essentially) in how they manipulate people’s desires. However, we’re then told that there is some evidence to the contrary; some advertisers are ethically motivated! That’s nice I guess, but I’m a bit skeptical; ‘ethical’ and ‘advertiser’ just don’t seem to go together often! Let’s see what evidence we have for this claim.

The argument cites a particular incidence where advertisers withdrew from a newspaper as it began to concentrate on sex and violence. Seems like the newspaper wants to profit from people’s desires! Our argument concludes the advertisers must have withdrew because they didn’t approve morally of the newspaper’s decision. This is the interpretation of the event which we want to support. What initially occurs to me is that it is entirely possible that the advertisers thought it would reflect badly on them if they were in the newspaper and could lose them money, and therefore their decision could have been entirely cynical. An answer choice which eliminates this alternate hypothesis that the advertisers were financially-motivated would be a good one. Let’s see what we get:

Answer Choice (A) Interesting, but this could be true if the advertisers withdrew for cynical reasons. This answer doesn’t give us enough information about the advertisers motives which is what we are interested in.

Answer Choice (B) We aren’t interested in those advertisers, we want to support a particular explanation of why some existing advertisers withdrew.

Correct Answer Choice (C) Bingo! If the advertisers knowingly took a haircut on their profits, that eliminates the alternate explanation that they did it all out of fear of losing money from not withdrawing.

Answer Choice (D) Ok? This, if anything, would lend support that the advertisers were motivated by financial rather than ethical considerations, and not strengthen our hypothesis.

Answer Choice (E) We don’t know whether this income group is poorer or richer and how that would affect the advertisers. Even if we did, how would it support our hypothesis that the advertisers withdrew because they were moral?

Take PrepTest

Review Results

Leave a Reply